SET 29 V Q 35: is any of the choices correct?

This topic has expert replies
Senior | Next Rank: 100 Posts
Posts: 36
Joined: Sat Jul 21, 2007 10:55 am
Location: India
Thanked: 1 times
Q35:
In Patton City, days are categorized as having heavy rainfall (more than two inches),
moderate rainfall (more than one inch, but no more than two inches), light rainfall (at
least a trace, but no more than one inch), or no rainfall. In 1990, there were fewer days
with light rainfall than in 1910 and fewer with moderate rainfall, yet total rainfall for the
year was 20 percent higher in 1990 than in 1910.
If the statements above are true, then it is also possible that in Patton City
A. the number of days with heavy rainfall was lower in 1990 than in 1910
B. the number of days with some rainfall, but no more than two inches, was the same
in 1990 as in 1910
C. the number of days with some rainfall, but no more than two inches, was higher in
1990 than in 1910
D. the total number of inches of rain that fell on days with moderate rainfall in 1990
was more than twice what it had been in 1910
E. the average amount of rainfall per month was lower in 1990 than in 1910

will give the OA later on...pl. explain ur ans...thanx;
Hi friends ! Just started GMAT prep. Hope to share thoughts, experience, joy and agony of GMAT prep journey.

Regards; neeraj

Master | Next Rank: 500 Posts
Posts: 460
Joined: Sun Mar 25, 2007 7:42 am
Thanked: 27 times

by samirpandeyit62 » Sat Nov 03, 2007 3:16 am
My undertanding here is that if total rainfall in 1910 was x then in 1990 it was 1.2x with fewer days of moderate & light rainfall as compared to that in 1910. (howver total rainfall would utilmately depend upon the total inches of rainfall rather than days)

so this is possible only if
a) the total haeavy rainfall in inches in 1990 was more than 1910

b) total moderate or light rf in inches in 1990 was more than 1910

now the Q asks
If the statements above are true, then it is also possible that in Patton City

so D is the choice which logically supports the arguement, as it says that total amt of moderate rainfall was more in 1990 than 1910 which may account for the 20% inc

what is the OA
Regards
Samir

Senior | Next Rank: 100 Posts
Posts: 36
Joined: Sat Jul 21, 2007 10:55 am
Location: India
Thanked: 1 times

by neeraj_99 » Sat Nov 03, 2007 4:19 am
the OA is not D...surprised? let's 've some more discussion...ur logic is fine...but why total inches of rf more than twice?...sud such exact conclusion can be drawn on the basis of the above info?
Hi friends ! Just started GMAT prep. Hope to share thoughts, experience, joy and agony of GMAT prep journey.

Regards; neeraj

Master | Next Rank: 500 Posts
Posts: 460
Joined: Sun Mar 25, 2007 7:42 am
Thanked: 27 times

by samirpandeyit62 » Sat Nov 03, 2007 4:34 am
Hi Neeraj,
U are right but the Q also asks abt a possiility & not an fact so thats why D seems possible to me.
Regards
Samir

Senior | Next Rank: 100 Posts
Posts: 64
Joined: Mon Nov 19, 2007 9:53 am
Thanked: 2 times

by jan08 » Fri Nov 23, 2007 9:53 am
here is my analysis:
A: Negate A coz the heavy rf stats are not discussed b/w 1990 and 1910
B: Negate B coz the no. of days with some rf (that includes moderate and light) is fewer is 1990 than in 1910...cannot be same
C: Negate C coz the no. of days with some rf (that includes moderate and light) is fewer is 1990 than in 1910...cannot be same..explanation same as B
D: Negate D coz the it is not sure that the moderate rainfall in 1990 is twice the moderate rainfall in 1910..

that leaves the answer E..logically it is also true...the no. of days of rainfall in 1990 is fewer than no. of days in 1910 and the amount of rainfall in 1990 is more than amount of rainfall in 1910..mathematically if u take the avg (no. of days/ total rainfall)...the avg for 1990 comes out to be less...I guess..

OA please?

Master | Next Rank: 500 Posts
Posts: 195
Joined: Sun Oct 21, 2007 4:33 am
Thanked: 10 times

by sankruth » Sun Dec 02, 2007 12:05 pm
IMO - A

Heres my (twisted!) analysis...

D - Not possible, because moderate rainfall on any day can never be twice of another day (Min possible is 1.1, Max possible is 2)

E - Not possible. Avg. rainfall per month = Total rainfall in year/12. Since 12 is constant, it only depends on total rainfall which we know is more for 1990 than 1910

B - Not true because
1990L less than 1910L
1990M less than 1910M
so, 1990(L+M) less than 1910(L+M).

Using the same logic for B, we can rule out C too.

So that leaves A

Senior | Next Rank: 100 Posts
Posts: 64
Joined: Mon Nov 19, 2007 9:53 am
Thanked: 2 times

by jan08 » Sun Dec 02, 2007 12:53 pm
Somebody please tell the OA with explanation..

Thanks

Junior | Next Rank: 30 Posts
Posts: 19
Joined: Thu Dec 13, 2007 6:45 am
Thanked: 1 times

by theroadrunnershow » Thu Dec 20, 2007 12:56 am
IMO A
eliminate E because avg=total/365 therefore not possible.
eliminate B and C because mentioned directly in the question.
eliminate D because moderate- 1 to 2 inches of rain. therefore there is no way in which the statement can be true.
Abhishek sunku

Master | Next Rank: 500 Posts
Posts: 222
Joined: Fri Jan 04, 2008 8:10 pm
Thanked: 15 times

by sibbineni » Mon Jan 21, 2008 3:50 pm
IMO A

I agree with sankruth

Legendary Member
Posts: 1578
Joined: Sun Dec 28, 2008 1:49 am
Thanked: 82 times
Followed by:9 members
GMAT Score:720

by maihuna » Sat Dec 19, 2009 12:49 pm
Isn't A an opposite answer? TestLuv can you explain, I am hacing C as possible answer. Here is my choice:

H: >2 1<M<2 trace<L<1 N
1910 1990
H
M more less
L more less
N
Total: Less More 20%
Opposite answer: A
Irrelevant: No info about average per month rainfall so E out, similarly D as total number of inches per day can not be determined....
Out of B and C: same is not a must, so C wins as it is possible that L+N is more in 90's.
Charged up again to beat the beast :)

GMAT Instructor
Posts: 1302
Joined: Mon Oct 19, 2009 2:13 pm
Location: Toronto
Thanked: 539 times
Followed by:164 members
GMAT Score:800

by Testluv » Sat Dec 19, 2009 2:53 pm
maihuna wrote:Isn't A an opposite answer? TestLuv can you explain, I am hacing C as possible answer. Here is my choice:

H: >2 1<M<2 trace<L<1 N
1910 1990
H
M more less
L more less
N
Total: Less More 20%
Opposite answer: A
Irrelevant: No info about average per month rainfall so E out, similarly D as total number of inches per day can not be determined....
Out of B and C: same is not a must, so C wins as it is possible that L+N is more in 90's.
The correct answer is definitely choice A. I discussed this problem in great detail here: https://www.beatthegmat.com/rainfall-pro ... 45-15.html (I posted on both pages 2, and 3 of the thread).
Kaplan Teacher in Toronto

Master | Next Rank: 500 Posts
Posts: 303
Joined: Sat Aug 22, 2015 10:23 am

by joseph32 » Sun May 15, 2016 10:29 pm
I will Go with option A in this case.